K
Khách

Hãy nhập câu hỏi của bạn vào đây, nếu là tài khoản VIP, bạn sẽ được ưu tiên trả lời.

24 tháng 5 2018

Xét hàm số là hàm số đồng biến trên R

AH
Akai Haruma
Giáo viên
31 tháng 10 2023

Đề thiếu. Bạn viết lại đề cẩn thận, rõ ràng để mọi người hỗ trợ tốt hơn bạn nhé.

25 tháng 12 2016

Vì x;y nguyên nên (2x-3)2 và |y-2| đều là số nguyên

Mà \(\hept{\begin{cases}\left(2x-3\right)^2\ge0\\\left|y-2\right|\ge0\end{cases}}\) nên (2x-3)2 và |y-2| là các số nguyên không âm

TH1: (2x-3)2=0 và |y-2|=1

\(\left(2x-3\right)^2=0\Leftrightarrow2x-3=0\Leftrightarrow2x=3\Leftrightarrow x=\frac{3}{2}\)(loại)

Ta không xét đến |y-2|=1 nữa!

TH2: (2x-3)2=1 và |y-2|=0

  • \(\left(2x-3\right)^2=1\Rightarrow\orbr{\begin{cases}2x-3=-1\\2x-3=1\end{cases}\Leftrightarrow\orbr{\begin{cases}2x=-2\\2x=4\end{cases}\Leftrightarrow}}\orbr{\begin{cases}x=-1\\x=2\end{cases}}\)
  • \(\left|y-2\right|=0\Leftrightarrow y-2=0\Leftrightarrow y=2\)

Vậy có 2 cặp x;y thỏa mãn là .........................

25 tháng 12 2016

\(!y-2!\le1\Rightarrow1\le y\le3\Rightarrow co.the=\left\{1,2,3\right\}\)

\(!2x-3!\le1\Rightarrow1\le x\le2=>x.cothe.=\left\{1,2\right\}\)

Với x=1,2=>có y=2

với 1,3 không có x thỏa mãn

KL:

(xy)=(1,2); (2,2)

8 tháng 4 2021

Áp dụng bđt bunhiacopxki, ta có:

\(\left(x^2+\frac{1}{x^2}\right)\left(1+16\right)\ge\left(x+\frac{4}{x}\right)^2\) => \(x^2+\frac{1}{x^2}\ge\frac{\left(x+\frac{4}{x}\right)^2}{17}\)

=> \(\sqrt{x^2+\frac{1}{x^2}}\ge\frac{x+\frac{4}{x}}{\sqrt{17}}=\frac{x}{\sqrt{17}}+\frac{4}{x\sqrt{17}}\)

CMTT: \(\sqrt{y^2+\frac{1}{y^2}}\ge\frac{y}{\sqrt{17}}+\frac{4}{\sqrt{17}y}\)

\(\sqrt{z^2+\frac{1}{z^2}}\ge\frac{z}{\sqrt{17}}+\frac{4}{\sqrt{17}z}\)

=> A \(\ge\frac{x+y+z}{\sqrt{17}}+\frac{4}{\sqrt{17}}\left(\frac{1}{x}+\frac{1}{y}+\frac{1}{z}\right)\ge\frac{x+y+z}{\sqrt{17}}+\frac{36}{\sqrt{17}\left(x+y+z\right)}\)(bđt: 1/a + 1/b + 1/c > = 9/(a+b+c)

=> A \(\ge\frac{16\left(x+y+z\right)}{\sqrt{17}}+\frac{36}{\sqrt{17}\left(x+y+z\right)}-\frac{15\left(x+y+z\right)}{\sqrt{17}}\)

\(\ge2\sqrt{\frac{16\left(x+y+z\right)}{\sqrt{17}}\cdot\frac{36}{\sqrt{17}\left(x+y+z\right)}}-\frac{15\cdot\frac{3}{2}}{\sqrt{17}}\)(Bđt cosi + bđt: x + y + z < = 3/2)

\(\ge\frac{48}{\sqrt{17}}-\frac{45}{2\sqrt{17}}=\frac{3\sqrt{17}}{2}\)

Dấu "=" xảy ra <=> x = y= z = 1/2

Vậy MinA = \(\frac{3\sqrt{17}}{2}\) <=> x = y = z = 1/2

10 tháng 11 2020

Bài 1: 

ĐK: \(x,y\ge-2\)

Ta có: \(\sqrt{x+2}-y^3=\sqrt{y+2}-x^3\Leftrightarrow\left(x-y\right)\left(x^2+xy+y^2\right)+\frac{x-y}{\sqrt{x+2}+\sqrt{y+2}}=0\)

=> x-y=0=>x=y

Thay y=x vào B ta được:  B=x2+2x+10\(=\left(x+1\right)^2+9\ge9\forall x\ge-2\)

Dấu '=' xảy ra <=> x+1=0=>x=-1 (tmđk)

Vậy Min B =9 khi x=y=-1

9 tháng 8 2020

10x100=

15 tháng 2 2017

ơ lạ ~ vì x;y;z đều là số dương nên x2<x5;y3<y6;z4<z7 cộng lại x2+y3+z4<x5+y6+zchứ, sao lại cho cái vế phải nhỏ hơn vế trái vậy???

15 tháng 2 2017

đề cho là số thực mà